Find the largest integer N such that a 2 b 2 c 2 + a b + b c + c a ≥ N a b c for all positive numbers a , b , and c .
This section requires Javascript.
You are seeing this because something didn't load right. We suggest you, (a) try
refreshing the page, (b) enabling javascript if it is disabled on your browser and,
finally, (c)
loading the
non-javascript version of this page
. We're sorry about the hassle.
By the AM-GM inequality, we have 4 a 2 b 2 c 2 + a b + b c + c a ≥ 4 a 2 b 2 c 2 ⋅ a b ⋅ b c ⋅ c a = a b c . Thus, N = 4 .
How do you know N can't be higher?
The question should be what is the least integer of N
N cannot be larger, since equality holds at some point in AM-GM (namely when a = b = c = 1 ).
Since i haven't learnt the AM-GM inequality yet,so i dun use it. however,a,b ,c can be any positive numbers .This results in the N will be also different. Let a=1,b=1,c=3,N will be 5.333333.
Log in to reply
It seems to me that your confusion stems from not understanding the question. Try answering this:
Find the largest integer N such that 4 ≥ N .
and compare it to
Find the smallest integer N such that 4 ≥ N .
What would be your answers?
The N has to hold for all a , b and c . If a = 1, b = 1 and c =3 then
1 2 × 1 2 × 3 2 + 1 × 1 + 1 × 3 + 3 × 1 = 1 6
and 1 6 \beq 4 × 1 × 1 × 3 = 1 2 ...
Log in to reply
however N can be 5 as well~^^ The question should ask us to find the smallest value of N.
Log in to reply
@Kelvin Ng – The problem is correct; it should be the largest N , because if it were any larger than 4, the inequality would not hold for ( 1 , 1 , 1 ) .
Log in to reply
@Aaron Doman – However,the question doesn't state that "a" should be equal to "b" and equal to "c".
Log in to reply
@Kelvin Ng – But the way it is phrased does not prohibit a , b , c being equal.
Log in to reply
@Aaron Doman – there are many possible groups of a,b and c....The question should ask the least value of N rather than the largest one because,there are many possible groups of (a,b,c).
Log in to reply
@Kelvin Ng – Let's try like this, give me the greatest number N that satisfies the inequality, so when i try every possible combination (a,b,c) it will hold. If you give me N>4, then I can find (a=b=c=1), so it means you were wrong. If you say the least, then N=1; N=2 and N=3, so there're infinite real answers so the problem won't make sense
We are interested in finding the minimum possible value of N, which turns out too be 4. so N may be greater than 4 as you found out. Try finding a,b,c such that N is less than 4.(You can't!!!)
I think that N can be any numbers as high as possible it can be.
Log in to reply
try a=b=c=1 and N=5
The problem is correct. The sticking point here is, as you basically figured out, the words "for all". It is not the same as "for some". We are looking for N that satisfy the given conditions even if the choice of a , b and c is left to our worst enemy (or a monkey with an unlimited access to a typewriter).
As a,b and c values increase,N can be also increased.
Using the AM-GM inequality, we have 3 a b + b c + c a ≥ 3 a 2 b 2 c 3 . So, we have:
a 2 b 2 c 2 + a b + b c + c a
≥ a 2 b 2 c 2 + 3 3 a 2 b 2 c 2 . . . . ( 1 ) .
So if N satisfies 1 , then it also satisfies the original inequality. If N satisfies 1 , then,
a 2 b 2 c 2 + 3 3 a 2 b 2 c 2 ≥ N a b c .
≡ p 4 − N p + 3 ≥ 0 where p = ( a b c ) 3 1 . Let f ( x ) = x 4 − N x + 3 . This function has a single real minima at x ⋆ = ( 4 N ) 3 1 . If the value of N is such that f ( x ⋆ ) ≥ 0 , then ( 1 ) is true for all N . Substituting x ⋆ in f ( x ) = 0 , and solving for N , we get N = 4 . It is easy to verify that the inequality a 2 b 2 c 2 + a b + b c + c a ≥ N a b c is not satisfied for N = 5 by letting a = b = c = 1 and noting that 1 + 3 ≱ 5 . So N = 4 is the largest integer that satisfies the inequality.
From AM-GM inequality
We know that AM ≥ GM Taking a 2 b 2 c 2 , a b , b c , c a as four terms we get
4 a 2 b 2 c 2 + a b + b c + c a ≥ 4 a 2 b 2 c 2 . a b . b c . c a
⇒ 4 a 2 b 2 c 2 + a b + b c + c a ≥ a b c
⇒ a 2 b 2 c 2 + a b + b c + c a ≥ 4 a b c
Thus N = 4
Using directly AM-GM inequality
4 a 2 b 2 c 2 + a b + b c + c a ≥ ( a 4 b 4 c 4 ) 4 1 = a b c
Therefore, a 2 b 2 c 2 + a b + b c + c a ≥ 4 a b c
Hence N = 4
Using A . M ≥ G . M inequality:
( a 2 b 2 c 2 + a b + b c + c a ) / 4 ≥ ( a 4 b 4 c 4 ) 1 / 4 or ( a 2 b 2 c 2 + a b + b c + c a ) ≥ 4 a b c
Comparing N = 4
Again, how do you know N can't be larger?
Log in to reply
I tried N=5 putting a,b,c as 1 which clearly do not satisfy.
Consider a=1, b=1, c=1.
1+1+1+1≥N.
N=4
How do you know there are no other values of a , b , c that result in a smaller value of N ?
Log in to reply
That's why the question should ask us to find the smallest value of N instead of the largest one.
Log in to reply
No, the question is fine as it is.
If the question was the smallest value of N then what would be you answer ???
By AM-GM Inequality,
a 2 b 2 c 2 + a b + b c + c a
≥ 4 4 \strut ( a 2 b 2 c 2 ) ( a b ) ( b c ) ( c a )
≥ 4 a b c
Hence, the maximum value of N is 4.
This can be done easily by using the AM-GM inequality
it is clear enough that if one puts any value less than 1 like a=0.2,b=0.5,c=0.9 etc......... n is bound to exceed 10. similar is the case with nos. greater than 1 where the value of n goes on increasing with succeeding inputs........ but when we put a=1,b=1,c=1 we deduce n=4........which happens to be the largest integer at which this expression holds good for all positive nos
We see that a, b and c are squared on the LHS which furthermore only contains terms increasing the value of the LHS. We see that a, b and c are not squared but linearly multiplied with each other on the RHS. This and the fact that a, b and c are positive integers means that we just need to minimize the values of a, b and c. This means we simply need to plug in ( a , b , c ) = ( 1 , 1 , 1 ) to arrive at the largest integer N. N = ( 1 2 ) ( 1 2 ) ( 1 2 ) + ( 1 ) ( 1 ) + ( 1 ) ( 1 ) + ( 1 ) ( 1 ) = 4
O menor valor que as incógnitas podem assumir é 1, daí,
1 2 ⋅ 1 2 ⋅ 1 2 + 1 ⋅ 1 + 1 ⋅ 1 + 1 ⋅ 1 ≥ N ⋅ 1 ⋅ 1 ⋅ 1 1 + 1 + 1 + 1 ≥ N 4 ≥ N N ≤ 4
Logo, N = 4
By AMGM, note
a b + b c > = 2 b a c
a b + a c > = 2 a b c
b c + a c > = 2 c a b
adding and dividing by 2 gives a b + b c + a c > = b a c + a b c + c a b substituting into the original equation and dividing by a b c a b c + 1 / a c + 1 / b c + 1 / a b > = N which clearly has a minimum when a = b = c = 1 hence N = 4
By AM-GM $$a^2b^2c^2 + ab + bc + ca \geq 4\sqrt[4]{a^4b^4c^4} = 4abc$$ On the other hand for ( a , b , c ) = ( 1 , 1 , 1 , 1 ) we get $$4 \geq N$$ Hence $$N = \boxed{4}$$
a^2 b^2 c^2 +ab+bc+ca >= Nabc.......Dividing each side of inequality by abc....................... As all three numbers a,b,c are positive, sign of inequality remains unchanged. Hence, abc+1/a+1/b+1/c>=N....Hence the greatest value of N is the least value of abc+1/a+1/b+1/c. Applyng AM- GM inequality, we get that abc+1/a+1/b+1/c>=4.......Hence , N max =4
a b c a 2 ∗ b 2 ∗ c 2 + a b + b c + c a = a b c + a + b + c
a b c + a + b + c > = N
Minimum condition for equation to be true: a,b,c = 1. 1 ∗ 1 ∗ 1 + 1 + 1 + 1 = 4 Therefore, largest integer * N = 4 *
The first step in your algebra is incorrect -- so even though you got the right solution, you should try this problem again.
On dividing with abc both sides we get::
abc + (1/a) + (1/b) + (1/c) >= N
Symmetrically a = b = c = 1 should be the solution
ANS :-: 4
what do you mean by "Symmetrically a = b = c = 1 should be the solution" as the question clearly states that a.b and c can take any value
Log in to reply
Based on maxima minima usually in such cases where there is symmetry there exists either a maxima or minima of the function
The symmetry argument is often good, but needs to be justified. For example, the triangle of a fixed perimeter with largest area is the symmetric solution, but the triangle of fixed perimeter with the smallest area is the non-symmetric degenerate case.
the largest number N for all positive numbers a , b , c , this mean a = b = c = 1 is included and it's the smallest possible solution for all ( a , b , c ) then:
a 2 b 2 c 2 + a b + b c + c a = ( 1 2 ) × ( 1 2 ) × ( 1 2 ) + ( 1 ) × ( 1 ) + ( 1 ) × ( 1 ) + ( 1 ) × ( 1 ) = 4
then 4 ≥ N so N = 4 is solution.
Problem Loading...
Note Loading...
Set Loading...
In the inequality, we let a = b = c = 1 to bound the possible values of N . This gives 4 ≥ N .
We now prove that N = 4 works; by AM-GM, we have
4 a 2 b 2 c 2 + a b + b c + c a ≥ 4 ( a 2 b 2 c 2 ) ( a b ) ( b c ) ( c a ) = a b c ,
so a 2 b 2 c 2 + a b + b c + c a ≥ 4 a b c for all positive a , b , c . Since 4 ≥ N and N = 4 is possible, the maximum is N = 4 .